Questions tagged [measure-theory]

Questions about abstract measure and Lebesgue integral theory. Also concerns such properties as measurability of maps and sets.

Filter by
Sorted by
Tagged with
2 votes
1 answer
70 views

$ \int_{E}^{*}{\psi (t) d\mu(t)}=\int_{E}{\phi (t) d\mu(t)} $

Let $(T, \mathcal{A}, \mu)$ be an arbitrary measure space. The outer integral over $(T, \mathcal{A}, \mu)$ of a (possibly nonmeasurable) function $\psi: T\to (-\infty, +\infty]$ is defined by: $$ \...
Karim KHAN's user avatar
3 votes
1 answer
212 views

A question about finitely additive integration

Let $(\Omega, \mathcal F, \mathbb P)$ be a probability space ($\mathbb P$ is countably additive). Let $\{p_\omega: \omega \in \Omega\}$ be a family of (countably additive) probability measures on $(\...
aduh's user avatar
  • 839
5 votes
1 answer
138 views

completeness of $\mathcal M(\Omega)$ without any topological assumptions?

Let $(\Omega,\Sigma)$ be a measurable space (no reference measure is chosen!), and $V$ a finite-dimensional normed vector space. Note carefully that I am not choosing any topology on $\Omega$, so the $...
leo monsaingeon's user avatar
1 vote
1 answer
165 views

Conditional expectation values defined by expectation values

I asked this question a couple of days ago on Math.SE but without any echo (no upvotes, although I offered a bounty). But because I did it for oversight from a reputable/professional source I now ...
Its_me's user avatar
  • 113
-1 votes
1 answer
88 views

Can we say that: $ \sum_{n\geq 1}{\frac{1}{n}(f_n(\omega)-g_n(\omega))}<\infty\qquad a.e $

Let $(\Omega,\mathcal{A},\mu)$ be a finite mesure space, and $\{f_n\}$ and $\{g_n\}$ two $L^1$-bounded sequences, such that : $$ \sum_{n\geq 1}{\frac{1}{n}(F_n(f_n)(\omega)-g_n(\omega))}<\infty\...
Parc John's user avatar
12 votes
1 answer
1k views

Riesz–Markov–Kakutani representation theorem for compact non-Hausdorff spaces

Let $X$ be a compact Hausdorff topological space, and $\mathcal C^0 (X) = \{f:X\to\mathbb{R}; \ f \text{ is continuous }\}$. It is well known that for any bounded linear functional $\phi: \mathcal C^...
Matheus Manzatto's user avatar
2 votes
2 answers
261 views

Existence of measurable "inclusion" into Euclidean space

Let $(\Omega,\mathfrak{F})$ be a measurable space. When does there exist an injective measurable function $f:(\Omega,\mathfrak{F})\to (\mathbb{R}^n,B(\mathbb{R}^n))$ to some Euclidean space, here $B(\...
ABIM's user avatar
  • 5,019
4 votes
2 answers
632 views

Disintegration, conditional probabilities, and conditional expectation

On the Wikipedia page there is a note that conditional probability measures can be described by disintegration. However, I can seem to find a clear exposée of how this construction is related to ...
ABIM's user avatar
  • 5,019
0 votes
1 answer
93 views

Can the joint law $P \circ (X,Y)^{-1}$ of two random variables $X$ and $Y$ be written as $P \circ (X,\phi(X,U))^{-1}$ for $U$ uniform in $[0,1]$?

I want to know whether there is some general assumpitons we can make on two measurable spaces $E$ and $F$ (e.g. polish, complete, separable,...) such that we can ensure that the following "Theorem" ...
vaoy's user avatar
  • 287
2 votes
0 answers
503 views

Example of a non-reflexive Banach space and two sequences

Let $(E,\mathcal {A}, \mu ) $ be a finite measure space and $X$ be a Banach space. The set of all Bochner-integrable functions from $E$ into $X$ is denoted by $\mathcal{L}_X^1$. If $X$ is reflexive, ...
Karim KHAN's user avatar
-3 votes
1 answer
452 views

Measure Theories with a different convention to $\infty\cdot 0 =0$ [closed]

As we all know in a first course in measure theory we define a symbol $\infty$ to satisfy $\infty \cdot 0=0$, but there are more two possible choices for a convention as someone has shown to me; one ...
Alan's user avatar
  • 1,514
-1 votes
1 answer
109 views

Isometric stratification preserves volume?

Let $K\subset \mathbb{R}^k$ be a non-empty compact subset let $f:K \to K$ be Lipschitz and surjective. If, moreover, $f$ is an isometry then clearly $f$ preserves the Lebesgue measure of $K$. I ...
ABIM's user avatar
  • 5,019
1 vote
2 answers
127 views

Locally compact Polish groups acting on standard Lebesgue spaces

If $G$ is a countable discrete group, then one can consider the Bernoulli shift $2^G$. $G$ acts on $2^G$ via shift, and letting $\mu$ be the product of the $(1/2, 1/2)$-measure in each coordinate, ...
Andy's user avatar
  • 369
2 votes
3 answers
173 views

Covering families of sets by small-measure partitions

Let $(X,\mathscr{A},\mu)$ be a probability space and let $\{A_1,\ldots,\}\subset\mathscr{A}$ be a countable family of sets with small measure: say $\mu(A_i)\le\epsilon$. I am trying to show that one ...
Aryeh Kontorovich's user avatar
2 votes
1 answer
542 views

Compact embedding of space of signed Radon measures into Sobolev space $W^{-1,q}$ from Evans paper; Does it work in one space dimension?

Background: I work on a PDE problem where I have some approximating sequence of measure-valued functions and I need to compactly embed it into some negative Sobolev space $W^{-m,q}$ on the bounded ...
Mark's user avatar
  • 647
0 votes
1 answer
79 views

difference between: Measurable multifunction integrably bounded and Measurable multifunction integrable

I read the article "Komlós Theorem for Unbounded Random Sets" by G. KRUPA (MSN), but I did not understand the difference between: Measurable multifunction integrably bounded, Measurable multifunction ...
kaka Hae's user avatar
  • 117
2 votes
3 answers
224 views

Image of probability measures under measurable mappings

Given two probability measures on two probability spaces, ($\mu, X$) and ($\gamma, Y$), what's the sufficient and necessary condition such that there is a measurable mapping $f:X\rightarrow Y$, such ...
user2173168's user avatar
0 votes
1 answer
77 views

Reduce ergodicity to the ergodicity of the coordinate process

Let $(E,\mathcal E,\lambda)$ be a probability space and $\lambda$ be a measurable map on $(E,\mathcal E)$ with $\lambda\circ\tau^{-1}=\lambda$. I would like to show that $\tau$ is $\lambda$-ergodic ...
0xbadf00d's user avatar
  • 161
3 votes
0 answers
203 views

Sets of finite perimeter: intersection with an half space

I have a question regarding sets of finite perimeter. In particular I'm interested to find $$\mu_{E \cap H_t}, \label{1}\tag{1}$$ where $E$ is a set of finite perimeter in a generic open set $\Omega \...
ty88's user avatar
  • 51
2 votes
1 answer
152 views

Atomless, c-additive measures in ZFC

This is a follow-up question to this one. Is there a ZFC example of an atomless measure that is $2^\omega$-additive, meaning, fewer than continuum many null sets have measurable union that is null?
Kant's user avatar
  • 55
3 votes
1 answer
199 views

Finitely additive, $\kappa$-additive atomless measures in ZFC

Under Martin's Axiom (and non-CH) the Lebesgue measure is $2^\omega$-additive in the sense that unions of fewer than continuum ($2^\omega$) many null sets are measureable and null. In ZFC we may ...
Kant's user avatar
  • 55
1 vote
0 answers
72 views

"Constructive" proof that compact sets $K\subseteq L_1(\mu\times\nu)$ are contained in products $S\widehat{\otimes} T$

A.Defant and K.Floret in chapter 7 of their Tensor Norms and Operator Ideals prove the equality $$ L_1(\mu\times\nu)\cong L_1(\mu)\widehat{\otimes}L_1(\nu) $$ for measures $\mu$ and $\nu$. At the same ...
Sergei Akbarov's user avatar
7 votes
1 answer
329 views

Prokhorov theorem on non Polish spaces

It is well known that if $X$ is a Polish space and $\mathcal{F} \subset \mathcal{M}_+(X)$ (the set of finite positive Radon measures on $X$) is uniformly tight and bounded in mass, it is relatively ...
Bremen000's user avatar
  • 327
2 votes
2 answers
314 views

Why the Komlós theorem is not valid for any sequence of measurable functions?

I read an article, and they use a certain theorem, called Komlós theorem, which says: Theorem 1 (Komlós theorem) Let $(E,\mathcal {A}, \mu ) $ be a finite measure space and $ (f_n)_{n\geq 1} \subset ...
Karim KHAN's user avatar
-2 votes
1 answer
931 views

Bounding $L^p$ norms in terms of lower-order $L^q$ norms

Suppose $f,g\in L^q(\Omega)$ ($\Omega\subset \mathbb{R}^n$) for all $1\le q\le p$. Here, $L^p(\Omega)$ is defined with respect to some measure $\mu$ that is absolutely continuous wrt Lebesgue measure. ...
JohnA's user avatar
  • 680
1 vote
1 answer
88 views

Convergence of probability measures which (asymptotically) concentrate along a submanifold

Let $V : (-1, 1)^d \to \mathbf{R}_+$ be a smooth function, and for $\beta > 0$, define \begin{align} P_\beta ( dx ) &= \exp \left( - \beta V ( x ) \right) / z (\beta) \, dx\\ z (\beta) &= \...
πr8's user avatar
  • 688
2 votes
0 answers
68 views

About the current of finite mass

In Demailly's e-book Complex analytic and differential geometry, chap3-(1.14) Proposition is stated as follows: Every positive current $T=i^{(n-p)^{2}} \sum T_{I, J} d z_{I} \wedge d \bar{z}_{J}$ ...
jack lion's user avatar
  • 391
1 vote
1 answer
164 views

Integrable functions as elements of closed absolutely convex hulls of precompact sets of indicator functions

I am not a specialist in measure theory, so excuse me if this is simple. Let $\mu$ be a finite measure on a set $X$ (for example, the Lebesgue measure on $[0,1]$). Integrable functions on $X$ can be ...
Sergei Akbarov's user avatar
2 votes
1 answer
147 views

Ergodic decomposition - how does restricting measure effect it? (Choquet Theory)

Suppose that $G$ is a discrete countable group and $\mu$ is an IRS (invariant random subgroup) of $G$: $\mu$ is conjugation invariant as a probability measure on the subgroups of $G$. Since all the $...
Mariah's user avatar
  • 181
5 votes
1 answer
399 views

When is the Radon-Nikodym derivative locally essentially bounded

Let $\mu\lll\nu$ be $\sigma$-finite Borel measures, which are not finite, on a topological space $X$. Under what conditions is $0<\operatorname{ess-supp}(\frac{d\mu}{d\nu}I_K)<\infty$ for every ...
ABIM's user avatar
  • 5,019
3 votes
1 answer
142 views

Density of $C(X,\operatorname{co}\{\delta_y\}_{y \in Y})$ in $C(X,\mathcal{P}(Y))$

Let $X,Y$ be locally-compact Polish spaces, equip the set $\mathcal{P}(Y)$ of probability measures on $Y$ with the weak$^{\star}$ topology (topology of convergence in distribution), and equip $C(X,\...
ABIM's user avatar
  • 5,019
0 votes
1 answer
334 views

Concentration of norm of linearly transformed normal random vector as dimension go to infinity

Earlier asked on MSE, but didn't get an answer, so posting here: Let $X=(X_1 \dots X_n) \in \mathbb{R}^n, X_i\sim N(0,1), iid.$ Let $B: \mathbb{R}^n \to \mathbb{R}^n $ be the diagonal linear map: $...
Learning math's user avatar
6 votes
1 answer
273 views

Comparing $X+Y$ and $X-Y$ for independent random variables with values in an abelian locally compact group

Let $G$ be an abelian locally (separable?) compact group with Haar measure $\mu$. Inspired by the interesting proof of A sum of two binomial random variables : Let $X$ and $Y$ be $G$-valued ...
Dieter Kadelka's user avatar
1 vote
1 answer
84 views

$ \|u_k-v_k\|_2\leq \min \bigg(\inf_{n\geq k}{\|f_n1_{\{|f_n|\leq k\}}\|_2},\frac{\epsilon_{k-1}}{4k}\bigg) $

Let $(E,\mathcal{A},\mu)$ be a finite measure space and $\{f_n\}_n$ be a sequence of simple functions such that: \begin{align*} f_n1_{\{\lvert f_n\rvert\leq k\}}\overset{\sigma(L^2,L^2)}{\underset{n}{\...
Kim Kim's user avatar
  • 13
1 vote
0 answers
170 views

Bounding the total variation metric between Gaussian mixtures

Let $\mathcal{P}(\mathbb{R}^d)$ the space of probability measures on $\mathbb{R}^d$ with total variation metric $\delta$, fix $k \in \mathbb{N}$, and let $\mathcal{P}'\subset \mathcal{P}(\mathbb{R}^d)$...
ABIM's user avatar
  • 5,019
3 votes
1 answer
206 views

A subadditive maximal ergodic theorem

Let $(\Omega,\mathcal A,\operatorname P)$ be a probability space, $\tau:\Omega\to\Omega$ be a measurable map on $(\Omega,\mathcal A)$ with $\operatorname P\circ\:\tau^{-1}=\operatorname P$, $Y_n:\...
0xbadf00d's user avatar
  • 161
5 votes
1 answer
233 views

How much time does a function spend above or below its average value around a point?

Given a locally integrable function $f: \mathbb R \to \mathbb R$, define $ K: \mathbb R \times \mathbb R+ \to \mathbb R$ by $$ K(x, r) := \begin{cases} 1, & \text{if }f(x) > \dfrac{1}{2r}\...
James Baxter's user avatar
  • 2,029
10 votes
2 answers
459 views

Graph metric approximating Euclidean metric

I've been reading Wolfram's recent articles about graph/mesh/grid structures as an analogy for physical space, and it seems to me that there will be a problem getting the notion of distance to work ...
Black Carrot's user avatar
18 votes
1 answer
510 views

Acting with a finite number of rotations on a set of positive measure can you fill almost the whole circle?

Let $E\subset S^1$ have positive Lebesgue measure. Do there exist finitely many rotations $r_1, r_2, \dots ,r_n$ such that $r_1E\cup r_2E\cup \dots\cup r_nE$ has measure $2\pi$? Or is there a ...
Claudio Rea's user avatar
0 votes
1 answer
54 views

Good upper-bound for $\mathbb E_A[e^{-t\|A\|_2}]$, for $t\ge0$ and random m by n matrix with iid entries with law $N(0,1)$

Let $A$ be a random $m$-by-$n$ matrix with iid $N(0,1)$ entries, $m$ and $n$ large with $n/m \longrightarrow \alpha \in (0, 1)$ . Let $\|A\|_2$ be the largest singular value of $A$ (i.e the spectral ...
dohmatob's user avatar
  • 6,716
6 votes
3 answers
549 views

Acting with all rational rotations on a subset of the circle having positive measure do you fill almost the whole circle?

Set $\Gamma$ for the group of the roots of the identity: $\Gamma=\{z\in \Bbb C | z^n=1$, for some $n\geq 0\}$ and for $E\subset S^1$ set $\Gamma E=\{z\zeta, z\in \Gamma, \zeta\in E \}$ A trivial but ...
Claudio Rea's user avatar
5 votes
1 answer
360 views

Inverse marginal property of a collection of $\sigma$-algebras

In my paper "On the inverse best approximation property of systems of subspaces of a Hilbert space" I introduced the Inverse marginal property (IMP) for a collection of $\sigma$-algebras. Let $(\...
Ivan Feshchenko's user avatar
2 votes
0 answers
114 views

Borel measurability

Suppose we have two locally compact Hausdorff spaces $X$ and $Y$. Let $i:X\to Y$ be a continuous injection. Under what condition the Borel $\sigma$-algebra of $X$ and $i(X)$ are isomorphic via the map ...
A beginner mathmatician's user avatar
0 votes
1 answer
110 views

Can we say that : $ \exists f_{\infty}\in L_{\mathbb{R}}^{1} \text{ such that: } f_n\to f_\infty\text{ a.e and in } L_{\mathbb{R}}^{1} $ [closed]

Let $(E,\mathcal{A},\mu)$ be a finite measure space and $\{f_n\}\subset L_{\mathbb{R}}^{1}$ such that: $$ \sum_{i=2}^{\infty}{\int_{E}{|f_n(t)-f_{n-1}(t)|d\mu(t)}}<+\infty $$ Can we say that : $$ \...
Made's user avatar
  • 115
4 votes
0 answers
189 views

Classification of Euclidean-invariant measures?

Is there a classification of measures on $\mathbb R^n$ which are invariant under (Euclidean) isometries? Hausdorff measures of all kinds are examples -- could that be all of them? More precisely, By ...
Tim Campion's user avatar
  • 60.6k
0 votes
1 answer
104 views

$\sum_{n=1}^{\infty}{\frac{1}{n^{1+\epsilon}}\mathbb{E}\big((|X_n|\mathbb{1}_{|X_n|\leq n})^{1+\epsilon}\big)}<\infty,~~\forall\epsilon>0 $

Let $(E,\mathcal{A},\mathbb{P})$ be a probability space $\{X_n\}$ be a sequence of random variable, such that: $$ (1)~.~~~\sup_n\mathbb E (|X_n|)<\infty\Rightarrow $$ $$ (2)~.~~~\dfrac{M_j}{2}<...
Made's user avatar
  • 115
3 votes
2 answers
328 views

Usefulness of the $\sigma(L^\infty,L^1)$ topology in the context of differential equations

In Brezis's Functional Analysis book through chapters 3-4, I've seen the $\sigma(L^\infty,L^1)$ topology on $L^\infty$ but did not see (so far) any application of it in differential equations. Is ...
UserA's user avatar
  • 587
4 votes
1 answer
471 views

Weak convergence in $L^1(X,\mu)$ space

I found an interesting property in some lecture notes on weak convergence: lemma 3.2 (2) on page 10: https://www.uio.no/studier/emner/matnat/math/MAT4380/v06/Weakconvergence.pdf . The idea is as ...
Maxim Diana's user avatar
1 vote
1 answer
143 views

The existence of a copy of a random variable with conditional expectation constraint

Let there be two random variables 𝑋 and 𝑌 with a certain joint copula. Is it always true that there is another random variable 𝑍 independent from 𝑌 such as the vectors $(X,Y)$ and $(X,Z)$ have the ...
Averroes's user avatar
  • 375
4 votes
1 answer
229 views

Is the intrinsic volume always positive for maximum dimension?

The intrinsic volume functions on $\mathbb{R}^d$ known from the Steiner formula and Hadwiger's Theorem can be extended to the domain of definable sets of an o-minimal structure $\text{Def}(\mathbb{R}^...
Joe Previdi's user avatar

1
18 19
20
21 22
58